Lower bound for the arithmetic mean based on quadratic mean

inequalitymeansupper-lower-bounds

It is known that the arithmetic mean of a list of non-negative real numbers is less than or equal to the quadratic mean (root mean square) of the same list:
$$\frac{x_1+x_2+\cdots+x_n}{n} \le \sqrt{\frac{x_1^2+x_2^2+\cdots+x_n^2}{n}}$$
(More about mean inequalities)

My question is that given the number $n$ (number of elements of the list) and the quadratic mean ($QM$) of that list can we find lower bound for the arithmetic mean ($AM$) of that list?

More precisely, what is the greatest lower bound for the $AM$ that we can find? For example, it is obvious that $AM \ge 0$. Also, it is easy to show that $AM \ge \dfrac{QM}{\sqrt{n}}$:
$$(x_1+x_2+\cdots+x_n)^2 \ge x_1^2+x_2^2+\cdots+x_n^2 \quad \Rightarrow\\
x_1+x_2+\cdots+x_n \ge \sqrt{x_1^2+x_2^2+\cdots+x_n^2} \quad \Rightarrow\\
\frac{x_1+x_2+\cdots+x_n}{n} \ge \frac{1}{\sqrt{n}} \sqrt{\frac{x_1^2+x_2^2+\cdots+x_n^2}{n}} \quad \Rightarrow\\
AM \ge \frac{QM}{\sqrt{n}}.$$

Is there some better (greater) lower bound for the $AM$ if we know $n$ and $QM$?

Best Answer

The bound $AM \ge \frac{QM}{\sqrt{n}}$ is tight. The equality is achieved, for instance, when one of $x_i$’s equals $1$ and the remaining equal $0$.